LSAT and Law School Admissions Forum

Get expert LSAT preparation and law school admissions advice from PowerScore Test Preparation.

User avatar
 KelseyWoods
PowerScore Staff
  • PowerScore Staff
  • Posts: 1079
  • Joined: Jun 26, 2013
|
#76048
Hi lw718!

Remember the stimulus tells us it is standard for all data to be corrected. So in the case of answer choice (B), both the data that conflicts with Jones's theory as well as the data that is in line with Jones's theory are corrected. It's just that the data that is in line with Jones's theory doesn't get subjected to the same amount of scrutiny. So basically the researchers have a bias where when their corrected data aligns with Jones's theory, they don't tend to question it, but when the corrected data doesn't align with Jones's theory, they scrutinize it more heavily, looking for more errors. This bias helps to explain the tendency of corrections to favor Jones's theory.

Hope this helps!

Best,
Kelsey
User avatar
 HEXUAN_UIBE
  • Posts: 1
  • Joined: Jan 07, 2022
|
#93148
Hi,
i want to share my understanding of this question.
firstly, we need to know where is the "discrepancy", it is very confusing and takes me a lot of effort to find it. but I finally found it.
actually, the discrepancy is the "majority", which means that most corrections make the collected data closer to the jones's theory. but why is this result? the normal situation should be that an equal amount of correction makes the collected data closer(the collected data is in conflict with the theory) as the correction makes the collected data no more closer(the collected data is in line with the theory).
finally, answer B explains this by indicating the researchers' greater scrutiny on the conflicting data.(is this familiar? i think a lot of easy questions use this method)
 Adam Tyson
PowerScore Staff
  • PowerScore Staff
  • Posts: 5153
  • Joined: Apr 14, 2011
|
#93195
Your reasoning looks good to me, HEXUAN_UIBE! The author tells us that there is this "striking tendency," which sounds like they find it odd. We need an answer that makes it seem normal, or at least understandable, why the corrected data tends to line up with Jones's theory more often than not.

I had two prephrases here:

1) Jones's theory is correct (which might explain why the correction of errors would lead to more data favoring that theory)
2) There is some bias among researchers in favor of Jones's theory, whether it is correct or not (which might lead researchers to intentionally or unintentionally bring data into line with the theory and to overlook errors in data that already lines up with it)

Answer B looks a lot like the second one about bias.

Add I agree with you about some of the easier Resolve questions: they do tend to be about discrepancies in reporting or in some scientific methodology, and the resolution to the paradox is that there is some bias or problem with the reporting or methodology. You will likely see more Resolve questions with a similar pattern!
User avatar
 cacurtis1
  • Posts: 2
  • Joined: Jan 08, 2021
|
#96682
I am confused on a basic understand of the argument as a whole.

WHAT EXACTLY IS JONE'S THEORY?

and can someone simply the last sentence (However....predicts.")

for some reason I cannot get a clear understanding of what the argument is saying.

greatly appreciated!!
User avatar
 katehos
PowerScore Staff
  • PowerScore Staff
  • Posts: 184
  • Joined: Mar 31, 2022
|
#96743
Hi cacurtis1!
 
cacurtis1 wrote: Thu Aug 11, 2022 11:48 am WHAT EXACTLY IS JONE'S THEORY?
We don't know! And, it doesn't matter (though I am curious :)).

We do know, however, what the argument is, so let's break it down to answer your question! The scientist starts off by telling us that data-collection is faulty ('errors can plague'), then the scientist tells us that researchers examine data to detect and subsequently correct data. In the last sentence, the scientist says that in the their field, these corrections seem to favor Jones' theory. The scientist then extrapolates upon this tendency to favor Jones' theory, saying that most of the corrected data conforms more with Jones' theory than the uncorrected version of the data. Basically, the scientist is saying it's odd that the corrected data favors Jones' theory!

Our task is to Resolve this Paradox. So, we must come up with an answer that explains why the corrected data favors Jones' theory. Whether it's that Jones' theory is correct, the researchers are biased (answer choice B), or for some other reason, we can use our knowledge of the stimulus to attack this question!

I hope this helps!
Kate

Get the most out of your LSAT Prep Plus subscription.

Analyze and track your performance with our Testing and Analytics Package.